Difference between revisions of "2019 AMC 10A Problems/Problem 4"

(Solution)
(Solution)
Line 10: Line 10:
  
 
Before having picked <math>15</math> balls of one color, we could have chosen <math>14</math> red balls, <math>14</math> green balls, <math>14</math> yellow balls, <math>13</math> blue balls, <math>11</math> white balls, and <math>9</math> black balls, for a total of <math>75</math> balls. After this, we have to pick one more ball to guarantee that <math>15</math> balls of a single color have been drawn. Thus, our answer is <math>75 + 1 = 76 \implies \boxed{\textbf{(B)}}.</math>
 
Before having picked <math>15</math> balls of one color, we could have chosen <math>14</math> red balls, <math>14</math> green balls, <math>14</math> yellow balls, <math>13</math> blue balls, <math>11</math> white balls, and <math>9</math> black balls, for a total of <math>75</math> balls. After this, we have to pick one more ball to guarantee that <math>15</math> balls of a single color have been drawn. Thus, our answer is <math>75 + 1 = 76 \implies \boxed{\textbf{(B)}}.</math>
 +
 +
(Also explained by Rearragenment Theorem)
  
 
==See Also==
 
==See Also==

Revision as of 22:53, 9 February 2019

The following problem is from both the 2019 AMC 10A #4 and 2019 AMC 12A #3, so both problems redirect to this page.

Problem

A box contains $28$ red balls, $20$ green balls, $19$ yellow balls, $13$ blue balls, $11$ white balls, and $9$ black balls. What is the minimum number of balls that must be drawn from the box without replacement to guarantee that at least $15$ balls of a single color will be drawn$?$

$\textbf{(A) } 75 \qquad\textbf{(B) } 76 \qquad\textbf{(C) } 79 \qquad\textbf{(D) } 84 \qquad\textbf{(E) } 91$

Solution

Before having picked $15$ balls of one color, we could have chosen $14$ red balls, $14$ green balls, $14$ yellow balls, $13$ blue balls, $11$ white balls, and $9$ black balls, for a total of $75$ balls. After this, we have to pick one more ball to guarantee that $15$ balls of a single color have been drawn. Thus, our answer is $75 + 1 = 76 \implies \boxed{\textbf{(B)}}.$

(Also explained by Rearragenment Theorem)

See Also

2019 AMC 10A (ProblemsAnswer KeyResources)
Preceded by
Problem 3
Followed by
Problem 5
1 2 3 4 5 6 7 8 9 10 11 12 13 14 15 16 17 18 19 20 21 22 23 24 25
All AMC 10 Problems and Solutions
2019 AMC 12A (ProblemsAnswer KeyResources)
Preceded by
Problem 2
Followed by
Problem 4
1 2 3 4 5 6 7 8 9 10 11 12 13 14 15 16 17 18 19 20 21 22 23 24 25
All AMC 12 Problems and Solutions

The problems on this page are copyrighted by the Mathematical Association of America's American Mathematics Competitions. AMC logo.png